LSAT and Law School Admissions Forum

Get expert LSAT preparation and law school admissions advice from PowerScore Test Preparation.

 kristinaroz93
  • Posts: 160
  • Joined: Jul 09, 2015
|
#20495
"Seemingly inconsequential changes in sea temperature due to global warming.."

I can't seem to eliminate B no matter how hard I try. Could someone please explain the problem in depthly and clearly as possible and as to why choice C is better than B.


Thanks in advance!
 Steve Stein
PowerScore Staff
  • PowerScore Staff
  • Posts: 1153
  • Joined: Apr 11, 2011
|
#20502
Hi Kristen,

That's an interesting one. The author states that sea temperature changes that seem small may still lead to a decline in fish and bird populations:
  • A 2 degree rise in temperature can prevent the mixing of seawater.
  • This hurts phytoplankton by taking nourishment away.
  • This, in turn, hurts zooplankton, which feed on phytoplankton.
  • This, in turn, hurts the rest of the food chain, which feeds on zooplankton.
The question asks for the role of the statement that zooplankton feed on phytoplankton, which is to show that part of the chain of events that eventually lead to the decline in fish and bird populations. This is what correct answer choice (C) provides: that statement shows how temperature changes have an indirect effect on larger animals.

The issue with answer choice (B) is that the statement regarding zooplankton does not exemplify how seawater mix can affect feeding habits; instead, it is another premise, in addition to the one about seawater mix, that is used to support the conclusion.

I hope that's helpful! Please let me know whether this is clear—thanks!

~Steve
 kristinaroz93
  • Posts: 160
  • Joined: Jul 09, 2015
|
#20504
To me it seems like both a premise and an example all in one, that is where my confusion lies=/ Aren't the hypothetical scenarios of what will happen if the temperature rises just examples used to move the argument along?
 Emily Haney-Caron
PowerScore Staff
  • PowerScore Staff
  • Posts: 577
  • Joined: Jan 12, 2012
|
#20507
Hi Kristin,

I think the piece you are missing here is that B says it is intended to provide an example. You can definitely argue that, in practice, it is an example of how the vertical mixing of seawater affects feeding habits, but it was not intended as an example; that was not its purpose. Also keep in mind that you are not looking for a right answer, you are looking for the best answer. In this case, the real purpose and function of the statement is to function as a critical premise in making the argument. C is therefore the best answer. Hope that helps!

Get the most out of your LSAT Prep Plus subscription.

Analyze and track your performance with our Testing and Analytics Package.